LSAT and Law School Admissions Forum

Get expert LSAT preparation and law school admissions advice from PowerScore Test Preparation.

 Administrator
PowerScore Staff
  • PowerScore Staff
  • Posts: 8917
  • Joined: Feb 02, 2011
|
#64915
Complete Question Explanation

Strengthen. The correct answer choice is (E)

The question stem that follows the stimulus is a strengthen question, so we should look for the
answer choice which lends the most credibility to the letter’s argument—that the department is not
spending five times as much to perform the same duties. If we want to prephrase this answer, there
are two possible ways to strengthen this argument: either show that the department is not currently
spending five times as much, or the department is not performing the exact same duties.

Answer choice (A): The budgets of other departments are irrelevant to the question of whether or not
the Planning Department is spending five times as much to perform the same duties, so this answer
choice does nothing to bolster the argument presented in the stimulus.

Answer choice (B): Since we are provided with no specific information about the department’s
various areas of expenditure, this choice does nothing to strengthen the author’s argument.

Answer choice (C): Since the relevant comparison is between the 2001 budget and this year’s budget,
this information is irrelevant to the argument.

Answer choice (D): Even if there were such adjustments, the figures cited still show the current
year’s expenditures at over five times as much as those of 2001, so this choice does not strengthen
the argument in the stimulus.

Answer choice (E): This is the correct answer choice. If a piece of 2003 legislation expanded the
duties of the Planning Department, then this strengthens the argument from the stimulus that the
department is not spending five times as much on performing the same duties.
 htngo12
  • Posts: 40
  • Joined: May 19, 2016
|
#27246
Hi!

For this Strengthen question:

Premise: Planning Budget increased from $100,000 in 2001 to $524,000 this year.
Conclusion: It is not justified that the department spends 5x as much money it did in 2001 to perform the same duties.

So I need to Strengthen the gap based on the amounts given that they did not spend as much $ now for the same duties, right?

I originally picked (D) as an answer. My thought process was that if the original budget was adjusted for inflation then
then it would explain the department did not spend 5x the money.

However, the correct answer is (E) A restructuring act, passed in 2003, broadened the duties of the planning Department.

I can see based the correct answer explains even with the amount of increase in 2001 to this year, the newly implemented act increased the duties of the Department so therefore the Department did not spend 5x times the amount for the same duties in 2001. (because it increased the duties).

I'm still missing the connection why (E) is the correct answer.
 Clay Cooper
PowerScore Staff
  • PowerScore Staff
  • Posts: 241
  • Joined: Jul 03, 2015
|
#27259
Hi htngo,

Thanks for your question.

You touched on the reason why answer choice E is correct. Our conclusion states that it is not true to say that the department used five times as much money this year to perform the same duties as it performed in 2001.

We can set the dollar amounts entirely aside; when we find out, in answer choice E, that the department performed different duties this year than it did it 2001, that is all we need to know to greatly strengthen our conclusion: it can't be the case that the department used much more money this year for the same duties it performed years ago if its duties now are broader.

Set the dollar amounts aside, and don't be distracted by them: the changing duties are enough to strengthen our conclusion.

I hope that helps!
 lsatstudent
  • Posts: 7
  • Joined: May 29, 2016
|
#27714
HI,

I also chose D over E, and while I understand why E is correct, can you please explain why D is incorrect.

Is the increase in the budget too high that it cannot be explained by inflation? :-?
 David Boyle
PowerScore Staff
  • PowerScore Staff
  • Posts: 836
  • Joined: Jun 07, 2013
|
#27741
lsatstudent wrote:HI,

I also chose D over E, and while I understand why E is correct, can you please explain why D is incorrect.

Is the increase in the budget too high that it cannot be explained by inflation? :-?

Hello lsatstudent,

Maybe that's precisely the case, i.e., that the budget increase is so large that even despite inflation, it increased five times over the old budget. Or maybe not; maybe with inflation, it only increased two times. The thing is, we don't know, so answer E is better.

Hope this helps,
David
 brcibake
  • Posts: 55
  • Joined: Jul 19, 2017
|
#40278
Clay Cooper wrote:Hi htngo,

Thanks for your question.

You touched on the reason why answer choice E is correct. Our conclusion states that it is not true to say that the department used five times as much money this year to perform the same duties as it performed in 2001.

We can set the dollar amounts entirely aside; when we find out, in answer choice E, that the department performed different duties this year than it did it 2001, that is all we need to know to greatly strengthen our conclusion: it can't be the case that the department used much more money this year for the same duties it performed years ago if its duties now are broader.

Set the dollar amounts aside, and don't be distracted by them: the changing duties are enough to strengthen our conclusion.

I hope that helps!
I eliminated E because it stated they performed different duties. The stimulus says it performed the same duties and so I don't see why E would be right,
 Adam Tyson
PowerScore Staff
  • PowerScore Staff
  • Posts: 5153
  • Joined: Apr 14, 2011
|
#40351
Be careful on this one, brcibake - the author is saying that they did NOT perform 5 times as much on the same duties! That was the claim in the editorial, but our letter to the editor is disputing that claim.

It's clear that the new budget is 5 times the old budget, regardless of any inflation or other factors. The amount of money spent (or at least budgeted to be spent) is not in dispute. That means the only thing that can be in dispute is the duties of the department. The editorial apparently said they are the same, and our letter writer must believe that they are not. To strengthen his case, we need to say that the duties have changed.

Take another look and see if that makes sense.
 k100
  • Posts: 10
  • Joined: Nov 21, 2019
|
#76672
After reading multiple explanations, I still can't seem to wrap my head around this!

Here's my thought process:
Argument breakdown: the budget increase doesn't justify the department to spend 5x more money to perform the same duties.

I don't see how (E) supports this claim (if anything, I thought it would weaken it) because I'm understanding it as "the department is not justified to spending more money after broadening their duties." Shouldn't it be the opposite? If they have more duties, shouldn't they be justified to have a bigger budget?

Thank you for your help!
 beeryslurs
  • Posts: 11
  • Joined: Mar 18, 2020
|
#76686
Hi. When the stimulus says "this year," does it refer to the year 2009, in which the PT was administered?
User avatar
 KelseyWoods
PowerScore Staff
  • PowerScore Staff
  • Posts: 1079
  • Joined: Jun 26, 2013
|
#76758
Hi k100!

Careful with how you're interpreting the argument here. The actual argument of the letter to the editor is that yesterday's editorial's conclusion that the Planning Department now spends five times as much money as it did in 2001 to perform the same duties is not justified.

Break it down this way:

The editorial said "The Planning Department now spends five times as much money as it did in 2001 to perform the same duties."

The letter to the editor says "The editorial's conclusion is not justified."

We're being asked to support the letter to the editor, which is attacking the editorial's conclusion. So essentially, to support the position of the letter writer, we need to attack the position of the newspaper.

Answer choice (E) does this by saying that the Planning Department has more duties now. Therefore, they may be spending 5 times as much as they were in 2001, but it's not to perform the same duties, it's to perform more duties. This would strengthen the letter to the editor's position that the editorial's conclusion was not justified.

It can be tricky to keep things straight when you're dealing with multiple speakers or, as in this case, with an argument which is arguing against another argument. So be sure to break it down carefully so you're clear on which position you're trying to support!

Hope this helps!

Best,
Kelsey

Get the most out of your LSAT Prep Plus subscription.

Analyze and track your performance with our Testing and Analytics Package.